You are on page 1of 16

Quant Updater – 14 | 17 June

Daily Quant Free Magazine for Bank Exam Students

17 June 2023 | SET - 14

Any Error or Doubt Found Please Mail us to :-


WhatsApp @ 8445784192
1
Quant Updater – 14 | 17 June
Daily Quant Free Magazine for Bank Exam Students

“जिसने संघर्ष को अपना साथी बना जिया


उसने ही इस दुजनया में
खुद को काबबि बना जिया.!!

मेरा नाम अरुण जसिं ह रावत हैं मैं साथियों जो BANK और Insurance Exams की परीक्षा देते है उन्हें Quant
विषय समझाने में मदत करता हूँ
यूँहा आज मैं आपसे एक मेंटर की तरह नहीं बल्कि एक दोस्त की तरह बात कर रहा हूँ !
मेरी Journey भी आपकी ही तरह ही है की हर पररिार का सपना उनका बेटा या बेटी एक सरकारी नौकरी करे
तो MATHS के डर ने काफी लोग को इस रेस में हमेशा पीछे ही रखा है इस साल मैंने कोशशश की है हर बच्चे को
बैंक की तैयारी के थलए बैंक एग्जाम की फीस के बराबर ही कोसस फीस देनी पड़े ले वकन क्लास के बाद प्रैल्किस
करने के थलए Quality Content जब बच्चो को थमल नहीं पा रहा िा तो तो सोचा की इस पर भी काम करते है
और सारे बच्चों के थलए एक फ्री मैगज़ीन तैयार की है शजसका नाम है " QUANT UPDATER "
दोस्तों आज भी अपने भारत में ऐसे बहुत सारे बच्चे है जो कोई कोसस या बैच खरीद नहीं पाते है अरुण सर ने उनके
थलए मात्र 199 रुपए में अपना एक कोसस लॉन्च वकया है आप उस कोसस की मदद से अपना फाउं डेशन अपना बेस
तैयार कर सकते है | दोस्तों बहुत अच्छा लगता है जब कोई स्टूडेंट्स मैसेज करता है वक सर आपकी गाइडन्स की
िजह से मेरा इग्ज़ैम हो गया है या फफर मैंने मैथ्स में 35 में 35 स्कोर वकया है ये जानकर अच्छा लगता है की मेरी
की हुई मेहनत का फल लोगों को थमल रहा है और उसकी सक्सेस का एक छोटा सा रहस्सा मैं भी हूँ !
इसी सोच को नया आयाम देने के थलए मैं ले कर आया हु जो की एक मैगजीन है " Quant Updater" शजसमे
आपको रोजाना सभी Quant Topic जैसे Simplification, Approximation, Number series, Data
interpretation aur Arithmetic के question प्रैल्किस करने के थलए थमले गा | इसके अलािा आपको
Detailed Solution फदया गया है जो आपकी प्रॉब्ले म को और आसान कर देगा |

Dosto is magazine ki help se aap apni speed improve kar sakte ho aur apne quant ke dar
ko hara sakte ho. ab bhi koi result aaye aap apne sapne ko pura hota dekhe.

अगर आपको कोई DOUBT रहता है वकसी सिाल पे तो आप मुझे मेल कर सकते है
Mathsbyarunsir007@gmail.com

दोस्तों आप सभी लोगों को एक नौकरी थमल जाए बस इसी सपने के साि आप मेहनत करो मैं हमेशा आपके
साि हूँ | दोस्तों शसले क्शन के बाद इं तजार रहेगा आपके एक मैसेज की सर मैंने कर फदखाया ! और आज मैं ने
अपने माूँ बाप और आपका नाम रोशन कर फदया है

Reasoning, English GA आप PRACTICE कर िो | Maths जसर्ष अरुण सर को कर िो | |

आपका दोस्त | आपका मेंटर


अरुण शसिं ह राित
( Maths by Arun Sir )
8445784192

Any Error or Doubt Found Please Mail us to :-


WhatsApp @ 8445784192
2
Quant Updater – 14 | 17 June
Daily Quant Free Magazine for Bank Exam Students

CONTENT
INDEX

Chapter – 1 | Speed Math - 2 Questions – 4 Minutes

1. Question Paper …………………………….. 4

2. Solution ……………………. 11

Chapter – 2 | Data Interpretations Graph Based - 4 Questions – 6 Minutes

1. Question Paper …………………………….. 5

2. Solution ……………………. 11

Chapter – 3 | Caselet Data Interpretation - 3 Questions – 6 Minutes

1. Question Paper …………………………….. 6-7

2. Solution ……………………. 11-14

Chapter – 4 | Arithmetic Based Questions - 7 Questions – 14 Minutes

1. Question Paper …………………………….. 8-10

2. Solution ……………………. 14-16

Any Error or Doubt Found Please Mail us to :-


WhatsApp @ 8445784192
3
Quant Updater – 14 | 17 June
Daily Quant Free Magazine for Bank Exam Students

Chapter – 1 | Speed Math

1. In the following approximation question, find which of the following statement/statements is/are true.
थनम्नथलखखत सल्किकटन प्रश्न में, ज्ञात कीशजए वक थनम्नथलखखत में से कौन सा/से किन/किन सत्य है/हैं।
{25.052 + 1799.98 ÷ 24.01 × 11.01} ÷ (x + 33.65) = 274.78 ÷ 10.89
I. Value of x lies between -24 to 25
II. x is an even number and less than the root of equation a2 – 42a + 416 = 0
III. x is a composite number less than 24.
I. x का मान -24 से 25 के बीच है
II. x एक सम संख्या है और समीकरण a2 – 42a + 416 = 0 के मूल से छोटा है
III. x 24 से कम एक संथमशित संख्या है।
(a) Only I (b) Both I and II (c) All the three (d) None of the three (e) Both I and III

2. Given below are two series I and II, having some missing terms (p), (q). Answer the question that follows.
Series I: 4, 7, 12, 19, (p), 43
Series II: 480, 96, (q), 8, 4, 4
Ramesh and Mahesh alone can finish the work in (p) days and (q) days respectively. Ramesh and Mahesh
together started working and work for 10 days, then what is the total time taken by Mahesh alone to finish the
remaining work?
नीचे दो शंखला I और II दी गई हैं , शजनमें कुछ लु प्त पद (p), (q) हैं । इसके बाद आने िाले प्रश्न का उत्तर दीशजए।
शंखिा I: 4, 7, 12, 19, (p), 43
शंखिा II: 480, 96, (q), 8, 4, 4
अकेले रमेश और महे श कायस को क्रमशः (p) फदनों और (q) फदनों में पू रा कर सकते हैं । एक साि रमेश और महे श ने कायस करना शुरू
वकया और 10 फदनों तक कायस वकया, तो अकेले महे श द्वारा शेष कायस को पूरा करने में थलया गया कुल समय वकतना है ?
(a) 4 days (b) 8 days (c) 6 days (d) 5 days (e) 9 days

Any Error or Doubt Found Please Mail us to :-


WhatsApp @ 8445784192
4
Quant Updater – 14 | 17 June
Daily Quant Free Magazine for Bank Exam Students

Chapter – 2 | Data Interpretations Graph Based


Graph Based
Directions (3 - 6): Given below is the table showing the investment of five persons in a business, time for which
investment made, share of profit and percentage of profit. Some values are missing in the table, you have to
calculate these value if necessary to answer the questions.
NOTE:- Profit percent is calculated on total profit made by all.
जनदेश (3 - 6): नीचे दी गई ताथलका में एक व्यिसाय में पांच व्यक्तियों का थनिेश , समय शजसके थलए थनिेश वकया गया, लाभ का रहस्सा
और लाभ का प्रवतशत फदखाया गया है। ताथलका में कुछ मान लु प्त हैं, यफद प्रश्नों का उत्तर देने के थलए आिश्यक हो तो आपको इन मानों
की गणना करनी होगी।
नोट:- लाभ प्रवतशत की गणना सभी द्वारा वकए गए कुल लाभ पर की जाती है।
Person Investment (in Rs.) Time (in month) Share of profit Percentage of profit

Aman 8 3600
%
211
Balram 20,000
Chinmay 10,000
Mayank 12 3600
%
211
Vaibhav 24,000 10,800

3. What is the sum of profit of Aman and Chinmay together if investment of Aman and Chinmay together is 215%
of investment of Balram and investment of Aman is 28% less than investment of Chinmay. Aman invested for
same time as Chinmay invested?
अमन और क्तचन्मय के एकसाि लाभ का योग वकतना है यफद अमन और क्तचन्मय का एकसाि थनिेश बलराम के थनिेश का 215% है और
अमन का थनिेश क्तचन्मय के थनिेश से 28% कम है। अमन ने उतने समय के थलए थनिेश वकया शजतना क्तचन्मय ने थनिेश वकया िा?
(a) 17200 (b) 18900 (c) 19400 (d) 14200 (e) None of these

4. What is the total profit of all 5 person if profit percentage of Varun is 50% more than profit % of Mayank?
यफद िरुण का लाभ प्रवतशत मयंक के लाभ% से 50% अक्तिक है , तो सभी 5 व्यक्तियों का कुल लाभ वकतना है ?
(a) 40500 (b) 43500 (c) 42200 (d) 53200 (e) 38500

5. What is the total investment of Chinmay and Varun if Varun invested for one month more than Chinmay and
ratio between the time taken by both i.e. Chinmay and Varun is 8 : 9?
यफद िरुण क्तचन्मय से एक महीने अक्तिक थनिेश करता है तो क्तचन्मय और िरुण का कुल थनिेश वकतना है और दोनों अिासत क्तचन्मय और
िरुण द्वारा थलए गए समय का अनुपात 8:9 है?
(a) 48000 (b) 47000 (c) 46000 (d) 49000 (e) 50000

350
6. Profit of Aman is what % of profit of Chinmay if profit of Chinmay is % of more than profit of Aman.
9
350
अमन का लाभ, क्तचन्मय के लाभ का वकतना% है यफद क्तचन्मय का लाभ अमन के लाभ से % अक्तिक है।
9
(a) 72% (b) 75% (c) 60% (d) 48% (e) 55%

Any Error or Doubt Found Please Mail us to :-


WhatsApp @ 8445784192
5
Quant Updater – 14 | 17 June
Daily Quant Free Magazine for Bank Exam Students

Chapter – 3 | Caselet Data Interpretation Based

Directions (7-9): Answer the questions based on the information given below.
Given below is the data regarding number of workers working in three different branches i.e. branch 1, branch
2 and branch 3 in four different factories i.e. ‘P’, ‘Q’, ‘R’ and ‘S’.
In factory ‘Q’: Number of workers working in branch 2 is 36% less than that in factory ‘P’. Number of workers
working in branch 1 is 624 which is 8(1/3)% more than that in factory ‘P’. Number of workers working in branch
2 is 20% more than that in branch 3.
In factory ‘R’: Number of workers working in branch 3 is 45% more than that in factory ‘Q’. Number of workers
working in branch 2 is 40% more than that in branch 1 and number of workers working in branch 3 is 16% more
than that in branch 1.
In factory ‘P’: Number of workers working in branch 2 is 24 more than that in branch 1 which is 60% more than
number of workers working in branch 3.
In factory ‘S’: Number of workers working in branch 2 is 37.5% less than that in factory ‘R’. Number of workers
working in branch 3 is 20% more than that in factory ‘P’ and total number of workers working in all three
branches is 1266.
जनदे श: नीचे दी गई जानकारी के आिार पर प्रश्नों के उत्तर दें ।
नीचे तीन अलग-अलग शाखाओ ं यानी शाखा 1, शाखा 2 और शाखा 3 में चार अलग-अलग कारखानों यानी 'P', 'Q', 'R' और 'S' में काम
करने िाले िथमकों की संख्या के संबंि में डे टा फदया गया है ।
फैक्ट्री 'Q' में: शाखा 2 में कायस रत िथमकों की संख्या फैक्ट्री 'P' की तुलना में 36% कम है। शाखा 1 में काम करने िाले िथमकों की संख्या
624 है जो फैक्ट्री 'P' से 8(1/3)% अक्तिक है । शाखा 2 में कायसरत कमसचाररयों की संख्या शाखा 3 की तुलना में 20% अक्तिक है ।
फैक्ट्री 'R' में: शाखा 3 में काम करने िाले िथमकों की संख्या फैक्ट्री 'Q' की तुलना में 45% अक्तिक है । शाखा 2 में कायसरत िथमकों की
संख्या शाखा 1 की तुलना में 40% अक्तिक है और शाखा 3 में कायसरत िथमकों की संख्या शाखा 1 की तुलना में 16% अक्तिक है ।
फैक्ट्री 'P' में: शाखा 2 में कायसरत िथमकों की संख्या शाखा 1 की तुल ना में 24 अक्ति क है जो शाखा 3 में कायसरत िथमकों की संख्या से
60% अक्तिक है ।
फैक्ट्री 'S' में: शाखा 2 में कायसरत िथमकों की संख्या फै क्ट्री 'R' की तुलना में 37.5% कम है । शाखा 3 में कायसरत िथमकों की संख्या
फैक्ट्री 'P' की तुलना में 20% अक्तिक है और तीनों शाखाओ ं में कायसरत िथमकों की कुल संख्या 1266 है ।

7. Ratio of number of workers working in branch 2 in factory ‘Q’ to number of workers working in branch 3 in
factory ‘R’.
फैक्ट्री 'Q में शाखा 2 में कायसरत िथमकों की संख्या का फैक्ट्री 'R' में शाखा 3 में कायसर त िथमकों की संख्या से अनुपात।
(a) 4:9 (b) 24:29 (c) 14:19 (d) 17:19 (e) None of these

8. Difference between total number of workers working in branch 2 in factory ‘P’ and ‘S’ together and number
of workers working in branch 2 in factory ‘R’.
फैक्ट्री 'पी' और 'एस' में शाखा 2 में काम करने िाले कमसचाररयों की कुल संख्या और फैक्ट्री 'आर' में शाखा 2 में काम करने िाले
कमसचाररयों की संख्या के बीच अंतर ज्ञात कीशजए।
(a) 390 (b) 345 (c) 375 (d) 385 (e) None of these

Any Error or Doubt Found Please Mail us to :-


WhatsApp @ 8445784192
6
Quant Updater – 14 | 17 June
Daily Quant Free Magazine for Bank Exam Students

9. If total number of workers working in factory ‘U’ is 75% more than that in factory ‘R’ and ratio of number of
workers working in branch 1 in factory ‘P’ and ‘U’ is 16:25, respectively and number of workers working in branch
2 in factory ‘U’ is 125% more than that in factory ‘Q’, then find the number of workers working in branch 3 in
factory ‘U’.
यफद फैक्ट्री 'U' में कायसरत िथमकों की कुल संख्या फैक्ट्री 'R' की तुलना में 75% अक्तिक है और फैक्ट्री 'P' और 'U' की शाखा 1 में कायसरत
िथमकों की संख्या का अनु पात क्रमश: 16:25 है और फैक्ट्री 'U' की शाखा 2 में कायसरत िथमकों की कुल संख्या फैक्ट्री Q की तुल ना में
125% अक्तिक है , तो फैक्ट्री 'U' की शाखा 3 में कायसरत िथमकों की संख्या ज्ञात कीशजए।
(a) 798 (b) 748 (c) 778 (d) 728 (e) None of these

Any Error or Doubt Found Please Mail us to :-


WhatsApp @ 8445784192
7
Quant Updater – 14 | 17 June
Daily Quant Free Magazine for Bank Exam Students

Chapter – 4 | Arithmetic Based Questions

10. Nandu invested Rs. ‘x’ for 2 years at 50% p.a. compound interest, compounded annually. He then lent the
interest received by him after 2 years to Kundan for one and half years at 40% p.a. compound interest,
compounded half yearly. After one and half years, Kundan returned Rs. (x + 800) to Nandu and invested the
remaining sum at 25% simple interest for 5 years. The interest received by Kundan after 5 years of investment is
Rs. 9440. Find the amount returned by Kundan to Nandu.
Nandu Rs 'x' को 2 िषों के थलए 50% प्रवतिषस चक्रिृद्धि ब्याज पर थनिेश करता है|2 िषस बाद िह प्राप्त ब्याज को Kundan को डेढ़ िषों के
थलए 40% प्रवतिषस छमाही देय चक्रिृद्धि ब्याज पर उिार देता है|डेढ़ िषों के बाद Kundan Nandu को Rs (x+ 800) िापस करता है और
शेष राशश को 5 िषों के थलए 25% सािारण ब्याज पर थनिेश करता है|थनिेश के 5 िषस के बाद Kundan द्वारा प्राप्त ब्याज Rs 9440 है|
Kundan, Nandu को वकतनी राशश िापस करता है?
(a) Rs. 7200 (b) Rs. 7900 (c) Rs. 6900 (d) Rs. 8800 (e) Rs. 8000

11. Time taken by Chetan to complete 65% of the whole work is 5 days more than time taken by Baldev to
complete 85% of the work which is 20 days less than time taken by Pawan to complete 60% of the work. If
Pawan and Baldev started the work and worked on it for 24 days, then find the time taken by Chetan to
complete the remaining work given that Pawan can complete the whole work in 90 days.
चेतन द्वारा पू रे कायस का 65% पूरा करने में थलया गया समय, बलदे ि द्वारा 85% कायस को पूरा करने में थलए गए समय से 5 फदन अक्तिक
है , जो पिन द्वारा 60% कायस को पूरा करने में थलए गए समय से 20 फदन कम है । यफद पिन और बलदे ि ने कायस शुरू वकया और उस पर
24 फदनों तक कायस वकया, तो शेष कायस को पूरा करने में चेतन द्वारा थलया गया समय ज्ञात कीशजए, फदया गया है वक पिन पूरे कायस को
90 फदनों में पूरा कर सकता है ।
(a) 4 days (b) 8 days (c) 10 days (d) 12 days (e) None of these

12. The question consists of two statements numbered “I and II” given below it. You have to decide whether
the data provided in the statements are sufficient to answer the question.
A shopkeeper had two mouse ‘A’ and ‘B’. No profit/loss is earned on selling mouse ‘A’ and discount given on
selling mouse ‘A’ is same as the price by which mouse ‘B’ is marked above its cost price. Find the selling price
of mouse ‘B’.
Statement I: Mouse ‘A’ is marked 40% above its cost price and selling price of mouse ‘A’ is Rs. 400 while
mouse ‘B’ is sold after two successive discounts of 25% and 20%, respectively.
Statement II: Marked price of mouse ‘A’ is 180% more than cost price of mouse ‘B’. Profit earned on selling
mouse ‘B’ is Rs. 16.
प्रश्न में दो किन "I और II" फदए गए हैं । आपको यह तय करना है वक किनों में फदया गया डे टा प्रश्न का उत्तर दे ने के थलए पयासप्त है या
नहीं।
एक दुकानदार के पास दो माउस 'A' और 'B' िे। माउस 'A' को बेचने पर कोई लाभ/हाथन नहीं होती है और माउस 'A' को बेचने पर दी
जाने िाली छूट उतनी ही होती है शजतनी कीमत पर माउस 'B' को उसके क्रय मूल्य से ऊपर अंवकत वकया जाता है । माउस 'B' का विक्रय
मूल्य ज्ञात कीशजए।

Any Error or Doubt Found Please Mail us to :-


WhatsApp @ 8445784192
8
Quant Updater – 14 | 17 June
Daily Quant Free Magazine for Bank Exam Students

कथन I: माउस 'A' को उसके लागत मूल्य से 40% अक्तिक अंवकत वकया गया है और माउस 'A' का विक्रय मूल्य 400 रुपये है जबवक
माउस 'B' को क्रमशः 25% और 20% की दो क्रथमक छूट के बाद बेचा जाता है ।
कथन II: माउस 'A' का अंवकत मूल्य माउस 'B' के लागत मूल्य से 180% अक्तिक है । माउस 'B' को बेचने पर अशजि त लाभ 16 रुपये है ।
(a) The data in statement I alone are sufficient to answer the question, while the data in statement II alone
are not sufficient to answer the question.
(b) The data in statement II alone are sufficient to answer the question, while the data in statement I alone
are not sufficient to answer the question.
(c) The data either in statement I alone or in statement II alone are sufficient to answer the question.
(d) The data given in both statements I and II together are not sufficient to answer the question.
(e) The data in both statements I and II together are necessary to answer the question.

13. The question consists of two statements numbered “I and II” given below it. You have to decide whether
the data provided in the statements are sufficient to answer the question.
There are two mixtures ‘A’ and ‘B’ containing rice and stone particles in it. Total quantity of stone particles in
both the mixtures together is 522 gm. Find the difference between quantity of stone particles in mixture ‘A’
and quantity of rice in mixture ‘B’.
Statement I: If 40% of mixture ‘A’ is taken out and is mixed with mixture ‘B’, then total quantity of mixture ‘B’
becomes 546 gm.
Statement II: Ratio of quantity of rice to stone particles in mixture ‘B’ is 8:9. Quantity of rice in mixture ‘A’
is 240 gm.
प्रश्न में दो किन "I और II" फदए गए हैं । आपको यह तय करना है वक किनों में फदया गया डे टा प्रश्न का उत्तर दे ने के थलए पयासप्त है या
नहीं।
इसमें चािल और पत्थर के कणों िाले दो थमिण 'A' और 'B' हैं । दोनों थमिण में पत्थर के कणों की कुल मात्रा 522 ग्राम है । थमिण 'A' में
पत्थर के कणों की मात्रा और थमिण 'B' में चािल की मात्रा के बीच अंतर ज्ञात कीशजए।
कथन I: यफद थमिण 'A' का 40% थनकाल थलया जाता है और थमिण 'B' में थमला फदया जाता है , तो थमिण 'B' की कुल मात्रा
546 ग्राम हो जाती है ।
कथन II: थमिण 'B' में पत्थर के कणों से चािल की मात्रा का अनुपात 8:9 है । थमिण 'A' में चािल की मात्रा 240 ग्राम है ।
(a) The data in statement I alone are sufficient to answer the question, while the data in statement II alone
are not sufficient to answer the question.
(b) The data in statement II alone are sufficient to answer the question, while the data in statement I alone
are not sufficient to answer the question.
(c) The data either in statement I alone or in statement II alone are sufficient to answer the question.
(d) The data given in both statements I and II together are not sufficient to answer the question.
(e) The data in both statements I and II together are necessary to answer the question.

Any Error or Doubt Found Please Mail us to :-


WhatsApp @ 8445784192
9
Quant Updater – 14 | 17 June
Daily Quant Free Magazine for Bank Exam Students

14. Monthly expenditure of Vaibhav is 40% of monthly income of Krishna and monthly savings of Krishna is 40%
more than that of Vaubhav. Monthly expenditure of Krishna is Rs. 3776 which is 47.2% of monthly income of
Vaibhav. Which of the following statements given below are true according to the given information?
I. Monthly savings of Krishna is Rs. 5824 II. Monthly income of Krishna is Rs. 10000
III. Monthly expenditure of Vaibhav is Rs. 3840
िैभि का माशसक व्यय कृष्णा की माशसक आय का 40% है और कृष्णा की माशसक बचत िैभि की तुलना में 40% अक्तिक है। कृष्णा का
माशसक व्यय Rs. 3776 है, जो िैभि की माशसक आय का 47.2% है। दी गई जानकारी के अनुसार नीचे फदए गए किनों में से कौन-सा सत्य
है?
I. कृष्णा की माशसक बचत Rs. 5824 है। II. कृष्णा की माशसक आय Rs. 10000 है।
III. िैभि का माशसक व्यय R 3840 है।
(a) Only I (b) Only II (c) Only I and II (d) Only III (e) Only I and III

15. Ratio of cost price of badminton racket and shuttecock is 4:5, respectively. If Badminton racket is marked 80%
above its cost price and sold after giving a discount of 20% while shuttlecock is sold at a profit of 50%, then overall
profit earned is Rs. ____ given that cost price of Badminton racket is Rs. _____.
The values given in which of the following options will fill the blanks in the same order in which is it given to make
the statement true:
बैडथमिं टन रैकेट और शटलकॉक के क्रय मूल्य का अनुपात क्रमशः 4:5 है। यफद बैडथमिं टन रैकेट को उसके लागत मूल्य से 80% अक्तिक
अंवकत वकया जाता है और 20% की छूट देकर बेचा जाता है जबवक शटलकॉक को 50% के लाभ पर बेचा जाता है , तो बैडथमिं टन रैकेट का
लागत मूल्य _____ है, तो अशजि त कुल लाभ _____ है।
थनम्नथलखखत में से वकस विकल्प में फदए गए मान ररि स्थान को उसी क्रम में भरेंगे शजसमें यह किन को सत्य बनाने के थलए फदया गया
है:
I. 426, 400 II. 329.5, 300 III. 639, 600
(a) Only I (b) Only II (c) Only I and III (d) Only I and II (e) All I, II and III

16. There are two mixtures ‘A’ and ‘B’ of cement and sand. The sum of quantities of cement in mixturse ‘A’ and ‘B’
is 280 kg. When 50% of mixture ‘B’ is mixed with mixture ‘A’, then total quantity of mixture will become 410 kg. Ratio
of cement to sand in mixture ‘B’ is 5:4 and mixture ‘A’ contains 140 kg of sand. Find the total quantity of mixture ‘A’
initially.
सीमेंट और रेत के दो थमिण 'ए' और 'बी' हैं। थमिण 'ए' और 'बी' में सीमेंट की मात्रा का योग 280 वकलो है। जब थमिण 'बी' का 50% थमिण
'ए' के साि थमलाया जाता है , तो थमिण की कुल मात्रा 410 वकग्रा हो जाएगी। थमिण 'B' में सीमेंट का रेत से अनुपात 5:4 है और थमिण 'A' में
140 वकग्रा रेत है। प्रारंभ में थमिण 'A' की कुल मात्रा ज्ञात कीशजए।
(a) 160 kg (b) 240 kg (c) 320 kg (d) 480 kg (e) None of these

Any Error or Doubt Found Please Mail us to :-


WhatsApp @ 8445784192
10
Quant Updater – 14 | 17 June
Daily Quant Free Magazine for Bank Exam Students

1. Solution (a) Sum of profit of Aman & Chinmay = 7200 +


{25.05 + 1799.98 ÷ 24.01 × 11.01} ÷ (? + 33.65)
2
10,000 = 17,200
= 274.78 ÷ 10.89
{252 + 1800 ÷ 24 × 11} ÷ (? + 34) ~ 275 ÷ 11 4. Solution (c)
3 3600 5400
{625 + 825} ÷ (? + 34) ~ 25 Profit %of Varun = × = %
2 211 211
? + 34 ~ 1450 ÷ 25 5400
% → 10800
211
? ~ 58 – 34 1% →
10800×211
5400
? ~ 24
So, 100% → 2 × 211 × 100
→ 42200
2. Solution (c)
Series I:
5. Solution (d)
4+3=7
Let both take 8x and 9x moth.
7 + 5 = 12
So Chinmay take 8 month and Varun take 9
12 + 7 = 19
month
19 + 11 = 30 = p 10000 y×8
= where y is Chinmay’s
10800 24000×9
30 + 13 = 43
investment
Series II:
y = 25000
480 ÷ 5 = 96 Total of Chinmay and Varun = 49000
96 ÷ 4 = 24 = q
24 ÷ 3 = 8 6. Solution (a)
8÷2=4 Let profit of Aman = x
4÷1=4 25
x = 10,000
18
Part of work done by Ramesh and Mahesh x = 7200
10 10 7200
together in 10 days = ( ) + ( ) Required % = × 100 = 72%
30 24 10000
40 + 50
=
120

=
3
7. Solution (b)
4

Remaining work = 1 - ( ) =
3 1 For factory ‘P’:
4 4
1th Let number of workers working in branch 3
Time taken by Mahesh alone to finish of
4
1
is ‘x’.
the total work = 24 × ( ) = 6 days
4 Number of workers working in branch 1
= 1.60 × x = 1.6x
3. Solution (a)
Number of workers working in branch 1
Investment of Aman and Chinmay together 12
= × 624 = 576
13
= 215 × 200 = 43000
So, 1.6x = 576
Investment of Aman = 18000
Or, x = 360
Investment of Chinmay = 25000
Number of workers working in branch 3
Ratio in which profit between Aman and
= 360
Chinmay is shared
Number of workers working in branch 2
(18 × 8) : (25 × 8)
18×10,000 = 576 + 24 = 600
Profit of Aman = = 7200
25

Any Error or Doubt Found Please Mail us to :-


WhatsApp @ 8445784192
11
Quant Updater – 14 | 17 June
Daily Quant Free Magazine for Bank Exam Students
464
Total number of workers = 576 + 600 + 360 Number of workers working in branch 1 =
1.16
= 1536 = 400
For factory ‘Q’: Number of workers working in branch 2
Number of workers working in branch 1 = 1.40 × 400 = 560
= 624 Total number of workers = 464 + 400 + 560
Number of workers working in branch 2 = 1424
= 0.64 × 600 = 384 For factory ‘S’:
Number of workers working in branch 3 Number of workers working in branch 3
384
= = 320 = 360 × 1.20 = 432
1.20
Total number of workers = 624 + 384 + 320 Number of workers working in branch 2
= 1328 = 0.625 × 560 = 350
For factory ‘R’: Number of workers working in branch 1
Number of workers working in branch 3 = 1266 – 432 – 350 = 484
= 1.45 × 320 = 464 Number of workers working in

Number of workers working in

Factory Branch 1 Branch 2 Branch 3 Total number of workers

P 576 600 360 1536

Q 624 384 320 1328

R 400 560 464 1424

S 484 350 432 1266

Desired ratio = 384:464 = 24:29 = 360


Number of workers working in branch 2
8. Solution (a) = 576 + 24 = 600
For factory ‘P’: Total number of workers = 576 + 600 + 360
Let number of workers working in branch 3 = 1536
is ‘x’. For factory ‘Q’:
Number of workers working in branch 1 Number of workers working in branch 1
= 1.60 × x = 1.6x = 624
Number of workers working in branch 1 Number of workers working in branch 2
12
= × 624 = 576 = 0.64 × 600 = 384
13
So, 1.6x = 576 Number of workers working in branch 3
384
Or, x = 360 = = 320
1.20
Number of workers working in branch 3

Any Error or Doubt Found Please Mail us to :-


WhatsApp @ 8445784192
12
Quant Updater – 14 | 17 June
Daily Quant Free Magazine for Bank Exam Students

Total number of workers = 624 + 384 + 320 For factory ‘S’:


= 1328 Number of workers working in branch 3
For factory ‘R’: = 360 × 1.20 = 432
Number of workers working in branch 3 Number of workers working in branch 2
= 1.45 × 320 = 464 = 0.625 × 560 = 350
Number of workers working in branch 1 Number of workers working in branch 1
464
= = 400 = 1266 – 432 – 350 = 484
1.16
Number of workers working in branch 2 Number of workers working in
= 1.40 × 400 = 560
Total number of workers = 464 + 400 + 560
= 1424

Number of workers working in

Factory Branch 1 Branch 2 Branch 3 Total number of workers

P 576 600 360 1536

Q 624 384 320 1328

R 400 560 464 1424

S 484 350 432 1266

Desired difference = (600 + 350) – 560 Total number of workers = 576 + 600 + 360
= 950 – 560 = 390 = 1536
For factory ‘Q’:
9. Solution (d) Number of workers working in branch 1
For factory ‘P’: = 624
Let number of workers working in branch 3 Number of workers working in branch 2
is ‘x’. = 0.64 × 600 = 384
Number of workers working in branch 1 Number of workers working in branch 3
384
= 1.60 × x = 1.6x = = 320
1.20
Number of workers working in branch 1 Total number of workers = 624 + 384 + 320
12
= × 624 = 576 = 1328
13
So, 1.6x = 576 For factory ‘R’:
Or, x = 360 Number of workers working in branch 3
Number of workers working in branch 3 = 1.45 × 320 = 464
= 360 Number of workers working in branch 1
464
Number of workers working in branch 2 = = 400
1.16
= 576 + 24 = 600 Number of workers working in branch 2

Any Error or Doubt Found Please Mail us to :-


WhatsApp @ 8445784192
13
Quant Updater – 14 | 17 June
Daily Quant Free Magazine for Bank Exam Students

= 1.40 × 400 = 560 Number of workers working in branch 2


Total number of workers = 464 + 400 + 560 = 0.625 × 560 = 350
= 1424 Number of workers working in branch 1
For factory ‘S’: = 1266 – 432 – 350 = 484
Number of workers working in branch 3 Number of workers working in
= 360 × 1.20 = 432

Number of workers working in

Factory Branch 1 Branch 2 Branch 3 Total number of workers

P 576 600 360 1536

Q 624 384 320 1328

R 400 560 464 1424

S 484 350 432 1266

Number of workers working in branch 1 in Or, x = Rs. 7200


25
factory ‘U’ = × 576 = 900 Therefore, required amount returned by
16
Number of workers working in branch 2 in Kundan to Nandu = 7200 + 800 = Rs. 8000
factory ‘U’ = 2.25 × 384 = 864
Total number of workers working in factory 11. Solution (b)
‘U’ = 1.75 × 1424 = 2492 Time taken by Baldev to complete 85% of
Number of workers working in branch 3 in the work = 0.60 × 90 – 20 = 34 days
factory ‘U’ = 2492 – 900 – 864 = 728 Time taken by ‘Chetan to complete 65% of
the work = 34 + 5 = 39 days
10. Solution (e) Time taken by ‘Baldev’ to complete the
Amount received by Nandu after 2 years = x whole work = 34/0.85 = 40 days
× (1.5)2 = Rs. 2.25x Time taken by ‘Chetan to complete the
Sum given by Nandu to Kundan = 2.25x – x whole work = 39/0.65 = 60 days
= Rs. 1.25x Let total amount of work = 360 units (LCM
Amount to be returned by Kundan = 1.2x × (1 of 90, 40 and 60)
360
+
40 1.5 × 2
) Efficiency of Pawan =
200 90

Or, A = 1.25 × (1.2) = Rs. 2.16x


3 = 4 units per day
360
Amount invested by Kundan at simple Efficiency of ‘Baldev’ = = 9 units per day
40
360
interest = 2.16x – (x + 800) = Rs. (1.16x – 800) Efficiency of ‘Chetan’ = = 6 units per day
60
According to the question, Amount of work completed by ‘Pawan’ and
{(1.16x – 800) × 1.25 = 9440 ‘Baldev’ together in 24 days
Or, 1.45x = 10440 = 13 × 24 = 312 units

Any Error or Doubt Found Please Mail us to :-


WhatsApp @ 8445784192
14
Quant Updater – 14 | 17 June
Daily Quant Free Magazine for Bank Exam Students

Remaining work = 360 – 312 = 48 units And, quantity of rice and stone particles in
Time taken by ‘Chetan’ to complete the mixture ‘B’ is ‘r’ gm and ‘s’ gm, respectively.
48
remaining work = = 8 days So, q + s = 522
6
Statement I:
12. Solution (e) 0.40 × (p + q) + (r + s) = 546
Statement I: So, data in statement I alone is not
Cost price of mouse ‘A’ = Selling price of sufficient to answer the question.
mouse ‘A’ = Rs. 400 Statement II:
r 8
Marked price of mouse ‘A’ = 1.40 × 400 =
s 9
= Rs. 560
And, p = 240
Discount given on selling mouse ‘A’
So, data in statement II alone is not
= 560 – 400 = Rs. 160
sufficient to answer the question.
Since, we do not have any information
Combining statement I and II, we get;
about marked price of mouse ‘B’. 8s
So, 0.40 × (240 + 522 – s) + ( + s) = 546
9
So, data in statement I alone is not 17s
Or, 304.8 – 0.4s + = 546
sufficient to answer the question. 67s
9

Or, = 241.2
Statement II: 45
Or, s = 162
So, data in statement II alone is not
8
So, r = × 162 = 144
sufficient to answer the question. 9

Combining statement I and II, we get; And, q = 522 – 162 = 360

Let cost price of mouse ‘B’ is Rs. ‘x’. So, difference between quantity of stone

So, marked price of mouse ‘B’ = Rs. (x + 160) particles in mixture ‘A’ and quantity of rice

Selling price of mouse ‘B’ in mixture ‘B’ = q – r = 360 – 144 = 216 gm

= 0.80 × 0.75 × (x + 160) = Rs. 0.6(x + 160) Therefore, the data in both statements I

So, 0.6(x + 160) – x = 16 and II together are necessary to answer the

Or, 0.6x + 96 – x = 16 question.

Or, 0.4x = 80
Or, x = 200 14. Solution (e)

So, selling price of mouse ‘B’ = 200 + 16 Monthly expenditure of Krishna = Rs. 3776
3776
= Rs. 216 Monthly income of Vaubhav =
0.472

Therefore, the data in both statements I = Rs. 8000


and II together are necessary to answer the Let monthly income of Krishna is Rs. ‘x’
question. Monthly expenditure of Vaubhav = 0.40 × x
= Rs. 0.4x
13. Solution (e) Let monthly savings of Vaubhav is Rs. ‘y’
Let, quantity of rice and stone particles in So, monthly savings of Krishna = 1.40 × y
mixture ‘A’ is ‘p’ gm and ‘q’ gm, = Rs. 1.4y
respectively. So, x – 1.4y = 3776.....................(1)
And, 0.4x + y = 8000....................(2)

Any Error or Doubt Found Please Mail us to :-


WhatsApp @ 8445784192
15
Quant Updater – 14 | 17 June
Daily Quant Free Magazine for Bank Exam Students

Solving (1) and (2), we get 5


Cost price of shuttlecock = ( ) × 300 = Rs. 375
4
x = 9600 and y = 4160 Selling price of Badminton racket
Monthly income of Krishna = x = Rs. 9600 = 0.80 × 1.80 × 300 = Rs. 432
Monthly savings of Krishna = 1.4y = 1.4 × 4160 Selling price of shuttlecock = 1.50 × 375
= Rs. 5824 = Rs. 562.5
Monthly expenditure of Vaubhav Overall profit earned
= 0.40 × 9600 = Rs. 3840 = 432 + 562.5) – (300 + 375) = Rs. 319.5
Monthly savings of Vaubhav = y = 4160 So, ‘II’ cannot be true.
For I: For ‘III’:
Since, Monthly savings of Krishna = Rs. 5824 Cost price of Badminton racket = Rs. 600
So, ‘I’ is true. 5
Cost price of shuttlecock = ( ) × 600
4
For ‘II’:
= Rs. 750
Since, Monthly income of Krishna = Rs. 9600
Selling price of Badminton racket
So, ‘II’ cannot be true.
= 0.80 × 1.80 × 600 = Rs. 864
For ‘III’:
Selling price of shuttlecock = 1.50 × 750
Since, Monthly expenditure of Vaubhav
= Rs. 1125
= Rs. 3840
Overall profit earned
So, ‘III’ is true.
= (864 + 1125) – (600 + 750) = Rs. 639
So, ‘III’ can be true.
15. Solution (c)
For ‘I’:
16. Solution (c)
Cost price of Badminton racket = Rs. 400
Let quantity of cement and sand in mixture
5
Cost price of shuttlecock = ( ) × 400 = Rs. ‘B’ is 5x kg and 4x kg respectively.
4

500 Quantity of sand in mixture ‘A’ = 140 kg


Selling price of Badminton racket Quantity of cement in mixture ‘A’
= 0.80 × 1.80 × 400 = Rs. 576 = (280 – 5x) kg
Selling price of shuttlecock According to the question,
= 1.50 × 500 = Rs. 750 0.5 × (5x + 4x) + 280 – 5x + 140 = 410
Overall profit earned Or, 4.5x – 5x = 410 – 280 – 140
= (576 + 750) – (400 + 500) = Rs. 426 Or, 0.5x = 10
So, ‘I’ can be true. Or, x = 20
For ‘II’: Therefore, total quantity of mixture ‘A’
Cost price of Badminton racket = Rs. 300 = 280 – 5x + 140 = 280 – 100 + 140 = 320 kg

PW Web/App - https://smart.link/7wwosivoicgd4

Library- https://smart.link/sdfez8ejd80if
For All Banking & Insurance Exam

Any Error or Doubt Found Please Mail us to :-


WhatsApp @ 8445784192
16

You might also like